A new test is developed to test for a certain allergy, giving "positive" or "negative" results to indicate that a person does or does not have the allergy. For a person who has the allergy, the test will give a positive result 82% of the time and a negative result (false negative) 18% of the time. For a person who does not have the allergy, the test will give a positive result (false positive) 10% of the time and a negative result 90% of the time. A study included a population of 10,000 people, and 1% of those studied where known to have the allergy. Fill in the missing values in the table below, and complete the statements that follow. Round the percentage to the nearest tenth, if necessary.

Answers

Answer 1

Answer:

the first blank is 82, the second blank is 8,910. I got that third blank wrong and it should be 7.1 I think. and the last block is should not. to find the third box all you need to do is:82+990=1,072 then divide 82 by 1,072 and round to the nearest tenth
Answer 2

Answer:

828,9107.6Should not


Related Questions

Find positive and negative coterminal
angles for pi over 6

Answers

Answer:

11n611  and -n6-n6

Step-by-step explanation:

How do I solve this problem?

Answers

Step-by-step explanation:

Note 12 is the difference between the numbers..

hope it helps

15 points, Trigonometric ratios. Please write your answers as it says on the mandate, Thanks!

Answers

Step-by-step explanation:

sec X = WX/XY = 37/12 = 3 1/12

or 3.083

PLEASE HELP ME WITH THIS

Answers

Answer:

A=1/2 ×1.0×2.4=1.2m² is a required answer

Answer:

1a) = 5

1b) = 4

2a)

A = 1/2 x 2.4 x 1.0

A = 1/2 x 2.4

A = 1.2 m²

Step-by-step explanation:

Find the area of the shaded sector. Round to the nearest hundredth

Answers

Answer:

54.98 sq. ft.

Step-by-step explanation:

The sector has an angle of 360 - 332 = 28

Area of the sector = 28π([tex]15^{2}[/tex])/360

                              = 19792.03372.../360

                              = 54.98 sq. ft.

If you buy a car for $349,985 and the car salesman gets 1/20 of the price of the car. How much will he get?

Answers

Answer:

$17499.25

Step-by-step explanation:

349,985*0.05=17499.25

Ruby started off her savings account with $100, and $0 in her checking account. She made $95 washing dishes and deposited it into her checking account. She bought a soda for $2 and then transferred $50 into her savings account. How much money is in each account now?

(Note: when you make purchases, like buying a soda, it comes out of your checking account)

A. She has $100 in savings and $95 in checking
B. She has $143 in her savings account and $50 in checking
C. She has $150 in savings and $43 in checking
D. She has $193 in savings and $0 in checking

Answers

Answer:

150+93=241

Step-by-step explanation:

I'm not sure if this is correct but i guess because she got 95 and deposited it into her checking she has 95 and judging on if she spent 2 dollars from her savings or checking i assumed it was savings which leaves it with 148 from 100 after depositing 50 - 2 = 148 so basically

edit: nvm i saw the note now im sure but something i dont understand is that if she made 95 how come the answer choices dont say 93 in checking so im guessing you meant 150+43=193

193 is your answer sorry lol took me a while to work this out or Answer choice C

An NBA fan named Mark claims that there are more fouls called on his team any other team, but the commissioner says that the number of fouls called against his team are no different than any other team. Mark finds that the average number of fouls in any game in the NBA is 11.5. He takes a random sample of 34 of games involving his team and finds that there are an average of 12.2 fouls against his team, with a standard deviation of 1.6 fouls. What is the correct conclusion

Answers

Answer:

Fouls > Average Fouls

Step-by-step explanation:

Null Hypothesis [ H0 ] : Fouls = Average Fouls ie 11.5

Alternate Hypothesis [ H1 ] : Fouls > Average Fouls

t =  ( x - u ) / ( s / √n )

where x = actual observed mean = 12.2 here , u = assumed population mean = 11.5 , s = sample standard deviation = 1.6 , n = sample size = 34

So, t = (12.2 - 11.5 ) / ( 1.6 / √34 ) = 0.7 /  ( 1.6 / 5.83 ) = 0.7 / 0.275 = 2.55

t tabulated value for α = 5% right tailed = 1.645 , ie < calculated value 2.55 So , we reject the null hypothesis, & conclude that 'Fouls > Average Fouls'

The area of the triangle below is 3.19 square inches. What is the length of the base?

Answers

Answer

m<a  is 90 and m<b is 40 i don't know which one is da base but i'd go with 40?

Step-by-step explanation:

Given the following equation, solve for x.
y = mx+b


Answers

Answer:

c

.......................

Does the question have no solution, one solution, or a infinite number of solutions?
2x + 3y = 6
2x + 3y = 7

Answers

Answer:

No solution

Step-by-step explanation:

I used this online calculator to find the answer. It shows the steps and everything.  https://www.symbolab.com/solver?or=gms&query=2x%2B3y%3D62x%2B3y%3D7

Answer:

no solution

Step-by-step explanation:

help please!!!!!! this is due soon

Answers

Answer:

lateral surface area of cone =πrl

132=22/7×r×11132×7=242rr=924/242r=3.81in

stay safe healthy and happy.

Please help me y’all!!

Answers

Answer:

d

Step-by-step explanation:

hope this helps

Here are two rectangular prisms:
K
L
6
5
4
5
5
5
1 Which figure do you think has the longer diagonal? Note that the figures are not drawn to scale
2. Calculate the lengths of both diagonals. Which one is actually longer?

Answers

Answer:

hey there buddy

Step-by-step explanation:

Please I need help with the question now PLEASE HELP

5. Marina lives in a 2-bedroom condominium overlooking a lake. Her expenses are as follows.
•The bi-weekly mortgage and property taxes are $718.
•The condo fees are $525 per month.
•The electricity bill averages $320 bi-monthly.
•The quarterly water bill averages $210.

a) Calculate Marina’s fixed monthly expenses for housing.

b) Calculate Marina’s average monthly expenses for utilities.

c) What is her average total monthly cost for housing?

Answers

Answer:

a) $1961

b) $710

c) $2671

Step-by-step explanation:

Bi-weekly mortgage & taxes: 718/2 weeks = 2 x 718 = $1436/month

Condo fees: $525/month

Electricity bill bi-monthly $320/2 weeks = 2 x 320 = $640/month

Water bill is quarterly, so 210/3 months. 210/3 = $70/month

a) housing expenses = mortgage, taxes, condo fees

1436 + 525 = $1961

b) utilities = electricity and water

640 + 70 = $710

c) all expenses listed

1961 + 710 = $2671

Gordon types 3,024 words in 42 minutes. Find the unit rate.

Answers

Answer:

72 words per minute

Step-by-step explanation:

[tex]\frac{1}{y}= \frac{42}{3024}[/tex]

42 × y = 1 × 3024

42y = 3024

42y ÷ 42 = 3024 ÷ 42

y = 72

Ava wants to draw a parallelogram on the coordinate plane. She
plots these 3 points.

Part A
Find and label the coordinates of the fourth vertex, K, of the
parallelogram. Draw the parallelogram.

Part B
What is the length of side JK? How do you know?​

Answers

Answer:

[tex]k =(2,1)[/tex]

[tex]JK = 2[/tex]

Step-by-step explanation:

Given

[tex]J = (0,1)[/tex] ---- [tex](x_1,y_1)[/tex]

[tex]H = (1,-2)[/tex] --- [tex](x_2,y_2)[/tex]

[tex]I = (3,-2)[/tex] --- [tex](x_3,y_3)[/tex]

See attachment for grid

Solving (a): The coordinates of K

The parallelogram has the following diagonals: IJ and HK

Diagonals bisect one another. So:

Midpoint of IJ = Midpoint of HK

This gives:

[tex]\frac{1}{2}(I + J) = \frac{1}{2}(H+K)[/tex]

[tex]\frac{1}{2}(x_3+x_1,y_3+y_1) = \frac{1}{2}(x_2+x,y_2+y)[/tex]

[tex]\frac{1}{2}(3+0,-2+1) = \frac{1}{2}(1+x,-2+y)[/tex]

[tex]\frac{1}{2}(3,-1) = \frac{1}{2}(1+x,-2+y)[/tex]

Multiply through by 2

[tex](3,-1) = (1+x,-2+y)[/tex]

By comparison:

[tex]1 + x = 3[/tex]

[tex]-2 + y = -1[/tex]

Solve for x and y

[tex]x = 3 - 1 =2[/tex]

[tex]y = -1 +2 = 1[/tex]

So, the coordinates of k is:

[tex]k =(2,1)[/tex]

The length of JK is calculated using distance (d) formula

[tex]d = \sqrt{(x_1 - x_2)^2 + (y_1 - y_2)^2[/tex]

[tex]J = (0,1)[/tex] ---- [tex](x_1,y_1)[/tex]

[tex]k =(2,1)[/tex] ---- [tex](x_2,y_2)[/tex]

So:

[tex]d = \sqrt{(0 - 2)^2 + (1 - 1)^2[/tex]

[tex]d = \sqrt{(- 2)^2 + (0)^2[/tex]

[tex]d = \sqrt{4 + 0[/tex]

[tex]d = \sqrt{4[/tex]

[tex]d = 2[/tex]

Hence:

[tex]JK = 2[/tex]

Round 2,034,627,105 to the nearest ten thousand.

Answers

Answer:

Heyaa!! My name is Pinky and i'm here to inform you that you're answer is...

Step-by-step explanation:

!!! 2,034,630,000 !!!

My calculations are:

You first look at the ten thousands place which is 27,105 and if you look to the right of the 2,

you see 7.5 and up round up 4 and down round down.

7 is 5 and up so, you round up to 2,034,630,000!!

Have a Nice Day!!

 ~Pinky~

CAN SOMEONE PLEASE HELP ME

WILL GIVE BRAINLIEST
( ANY LINKS OR WEBSITES WILL BE REPORTED)

Answers

Answer:

40

Step-by-step explanation:

80 divided by 2 is 40 and the formula is L times W times Height and the height is 2 so the base is 40 B times H

It's 40

A PE class has 48 students. One group of 24 students chooses to play volleyball. The teacher then divides the
remaining students into as many groups of 5 as possible to shoot baskets. After that the remaining students (not
on a volleyball or basketball team) climb the rock wall.
How many students climb the rock wall?
students

Answers

Subtraction is a mathematical operation that reflects the removal of things from a collection. The number of students who climbs the rock wall is 4.

What is subtraction?

Subtraction is a mathematical operation that reflects the removal of things from a collection. The negative symbol represents subtraction.

The total number of students in the class is 48, out of these 48 students, 24 students choose to play volleyball. Therefore, the remaining students are 24 (48-24).

Students remaining after volleyball

  = Total number of students - Number of students who play volleyball

  = 48 - 24 = 24

Now, The teacher then divides the remaining students into as many groups of 5 as possible to shoot baskets. Therefore, the teacher divides the remaining 24 students into 4 groups of 5 people. After this, the number of students that are remaining is 4(24-20).

Students remaining after basketball

  = Total number of students - Number of students who shoot basketball

  = 24 - (4x5)

  = 24 - 20 = 4

Further, the remaining students go for climbing the rock wall.

Hence, the number of students who climbs the rock wall is 4.

Learn more about Subtraction:

https://brainly.com/question/1927340

#SPJ1

The formula for centripetal acceleration, a, is given below, where v is the velocity of the object and r is the object's distance from the center of the circular path.

a= v^2/r

Solve the formula for the velocity.

v=

Answers

Answer:

Step-by-step explanation:

For this case we have the following equation:

a = v ^ 2 / r

From here, we must clear the value of speed.

We have then:

v ^ 2 = a * r

v = root (a * r)

Answer:

the formula for the velocity is:

v = root (a * r)

Answer:

v=[tex]\sqrt{ar}[/tex]

Step-by-step explanation:

What is the least common denominator of 1/2, 2/3, and 3/4?

48
24
16
12

Answers

Answer:

12

Step-by-step explanation:

This is because 2, 3, and 4 can all go into the number and it is the smallest of the ones listed.

Answer: 24

Step-by-step explanation:

1/2 x 12 = 12/24

2/3 x 8 = 16/24

3/4 x 6 = 18/24

Each number can be multiplied by "x" to get a denominator of 24.

Darla is creating a family tree. She starts by writing down the names of her two parents. For each of her parents, she writes down the names of their two parents, and so on. After six generations, how many names has she written down, NOT including herself?

Answers

6x2 right ...?????......

important question need help

Answers

Inthinknitsbthe second one I THINK

Answer:

options 3 and 4

Step-by-step explanation:

The exponential function f with base a is defined by

f(x) = a^x

so the exponential functions are options 3 and 4

How many inches are equal to 12 cm ?

Answers

Answer:

12/2.54 ≈ 4.72 inches

Answer: 4.72 inches

How?

The question gave us the formula: 2.54cm = 1in. So using that formula, we divide 12 by 2.54 (12/2.54).

12/2.54 = 4.72440945

Rounded to the nearest hundredth is

4.72.

What is the rate of Change in y=8x - 4

Answers

Answer:

The slope or rate of change is 8

Step-by-step explanation:

There isn't much to explain, sorry! I hope this helps!

PLS HURRY
Jaime rode her bike 5 miles north to the park to meet a friend. Then they rode east to the library. If Jaime's house is 13 miles from the library, about how far is the library from the park?

Answers

The distance from library and the park is 12 miles.

What is trigonometric Ratios?

"Trigonometric Ratios are defined as the values of all the trigonometric functions based on the value of the ratio of sides in a right-angled triangle. It is with respect to any of its acute angles are known as the trigonometric ratios of that particular angle".

For the given situation,

Jaime ride her bike in north = 5 miles

Jaime ride her bike in east = 13 miles

The distance from library and the park be x.

The below diagram shows about the Jaime's riding.

The distance from library and the park can be find by using Pythagoras theorem,

[tex]Hypotenuse^{2} = Perpendicular^{2} + base^{2}[/tex]

On substituting the above values,

⇒ [tex]13^{2} = 5^{2}+x^{2}[/tex]

⇒ [tex]x^{2} =169-25[/tex]

⇒ [tex]x^{2} = 144[/tex]

⇒ [tex]x=\sqrt{144}[/tex]

⇒ [tex]x=12[/tex]

Hence we can conclude that the distance from library and the park is    12 miles.

Learn more about trigonometric ratios here

brainly.com/question/9085166

#SPJ2

Carlos and Clarita were asked to write y=2x2+4x-6 in factored form.

What do you think?

Answers

Answer:

2(x+3)(x-1)

Step-by-step explanation:

Because 2x^2+4x-6 can be divided by 2, you can divide them by two. This would make the equation 2(x^2+2x-3). To factor this, you have to find two equation that would make it true. The equation have to be _+_=2 and _x_=-3. The two blanks would be 3 and -1. 3+-1=2 and 3x-1=-3. This means that the equation is 2(x-1)(x+3).

If you need an explanation tell me but if this answer helped you, pls mark as brainliest

Will give Brainliest!!!

In this passage, the phrase "territorial integrity"
indicates that the league should offer protection
O for free political elections.
O for all leaders in power.
O from all taxes and tariffs.
O from attacks by other nations.

Answers

Answer: D) From attacks by other nations

Step-by-step explanation:

Answer: D. from attacks by other nations.

Step-by-step explanation:   TRUST ME!!! Answer correct on Edge!  :)

Angela is buying a dress that is on sale for 20% off. If the original price of the dress is $40.00, how much money is Angela saving on the dress?
A. $0.80
B. $8.00
C. $3.20
D. $32.00​

Answers

Angela is saving $8.00 on the dress bc 20% of 40 is 8.
The answer isssss........ B
Other Questions
How do you solve this? What molecule(s) do plants release into the air? What industry was most influenced by the gold rush? TimberSeafood RailroadNatural gas PLS HELP NO LINKS THANK YOU ((((((The odds are too great)))))))----- what does it mean? explain it. PLEASE. What type of data distribution is shown on the graph? A bar graph has x on the x-axis, from 0 to 4, and P x (x) on the y-axis, from 0 to 0.5 in 0.1 increments. The probability of 0 is 0.01, the probability of 1 is 0.25, the probability of 2 is 0.3, the probability of 3, is 0.4, and the probability of 4 is 0.02. Keegan is building a rectangular play area in his back yard for his dog. The length of the rectangle is 30 feet and the width is 24 feet what amount of a fence does Keegan need to buy to make th play area. Proteins are used by the body forA.Water and OxygenB.Carbon Dioxide and GlucoseC.Water, Carbon Dioxide and EnergyD.Just energy what is the answer?17%20%14%1/51/61/7 HELP ASP!! DUE TONIGHTWhat stimulates the release of the enzyme amylase into the pancreatic duct so that it can break down starches?A. acetylcholine molecules from a nearby neuronB. the breakdown of glucose in the chloroplastC. the division of the cellD. the presence of protein in the stomachE. a decrease in the volume of the large intestine Which is equivalent to (4 x y minus 3 z) squared, and what type of special product is itIn which step did Fiona make an error?? Jina made 6 identical necklaces, each having beads and a pendant. The total cost of the beads and pendants for all 6 necklaces was $33.60. If the beads cost atotal of $14.40, how much did each pendant cost? Challenge A bag contains pennies, nickels dimes, and quarters. There are 50 coins in all of thecoins 18% are pennies and 40% are dimes. There are 2 more nickels than pennies. How muchmoney does the bag containThe bag contains S How does this picture attack the emotions and bring people over to buy warbonds? No tienesgasolina para viajar en tucoche; por eso vas________ autobs.A.) por: transportation B.) para: recipientC.) por: reason or motiveD.) para: purpose or goal Write a program that prompts the user to enter a series of numbers between 0 and 10 asintegers. The user will enter all numbers on a single line, and scores will be separatedby spaces. You cannot assume the user will supply valid integers, nor can you assumethat they will enter positive numbers, so make sure that you validate your data. You donot need to re-prompt the user once they have entered a single line of data. Once youhave collected this data you should compute the following: The largest value Thesmallest value The range of values The mode (the value that occurs the most often) A histogram that visualizes the frequency of each number Heres a sample running of theprogram. What is the value of B for the following triangular prism?6 yd 212 yd 217.5 yd 215 yd 2 Shakespeare wrote 7 plays in 4 days. At this rate, how many plays will he be able to write in 10 days? What is 24^2? Use mental math. Which three structural zones overlap with the mantle ?